If the surface area of a sphere is 484π, what is the radius?

Answers

Answer 1

Answer:

6.21 is the radius I believe

Step-by-step explanation:

Answer 2

Answer:

It would be 11.

Step-by-step explanation:

The equation for the surface area of a sphere is 4πr^2, so lets reduce the answer to get the value of r, the radius.

484π = 4πr^2

Divide by π

484 = 4r^2

Divide by 4

121 = r^2

Lastly, square root each side

11 = r

r is 11.


Related Questions

The radius of a circle is 4 feet. What is the circle's circumference?Use 3.14 for
r=4 ft

Answers

Answer:

50.24 is the circumfrence

If we were to use actual pi the answer would be -

50.26548246

Answer:

25.12

Step-by-step explanation:

2pi4=2*3.14*4=25.12

conviince me for my question (brainliest plzz)

A coin is flipped 160 times. It landed on heads 82 times and tails 78 times. A different coin is flipped 250 times. It landed on heads 115 times and tails 135 times. What is the empirical probability of flipping a heads on the first coin AND flipping a heads on the second coin

Answers

Answer:

a. first coin heads: 82/160

b. second coin heads: 115/250

a AND b: a x b = 82/160 x 115/250

Step-by-step explanation:

if not then im confuzzled

What is this shape? (sorry I'm d.umb)

Answers

Answer:

triangular prism

Step-by-step explanation:

Question 2
0.5 pts
Write 0.1133 as a fraction.
(Does not need to be simplified)
[a]
Question 3
0.5 pts
what

Answers

Answer:

1133/10000

Step-by-step explanation:

Hi how’s your day ? Would you like to help me ?

Answers

Answer:

Hi

Good

I think its the second but i'm not sure because there are no measurements

Good luck

Hi could some PLEASE help me !!!!

Answers

Answer:

3x=78+x-2

Step-by-step explanation:

HELP 30 POINTS please in a hurry
Is the relation shown below a function? Use the graph below to justify your response.
(0, 0), (2, 0), (2, 2), and (3, 4)

Answers

To be a function every input value (x) can only have one output value (y)

On the graph there are 2 out put values for x = 2 so it is not a function.

The answer would be C.

repost on my warm up plz help

Answers

Answer:

oh srry ur not a scam

Step-by-step explanation:

Answer:

#1: (Problem 3² × 3³, Answer: 243) (Problem 3⁵, Answer: 243), yes the answer matches

#2: (Problem 2³ × 2⁴, Answer: 128) (Problem 2¹², Answer: 4096), No, the answer doesn't match.

#3: (Problem (-2)⁴ × (-2)², Answer: 64) (Problem (-2)⁸, Answer: 256), No the answer doesn't match.

#4: (Problem (-3)² × (-3)⁵, Answer: -2187) (Problem (-3)⁷, Answer: -2187), yes the answer matches

The pattern is: When 2 whole numbers raised to a power are multiplied together, the base stays the same and the exponent add up

Hope this helps!

What are the measures of ∠1 and ∠2?

Answers

Answer:

Angle 1 = 102

Angle 2 = 40.9

Step-by-step explanation:

[tex]1)\ 180 - 121.8 = 58.2[/tex]

[tex]2)\ angle\ 2 = 58.2 - 17.3 = 40.9[/tex]

[tex]3)\ angle\ 1 = 180 - 40.9 - 37.1 = 102[/tex]

Answer:

m∠1  = 67.4°      m∠2 = 104.5°

Step-by-step explanation:

17.3 + m∠2 = 121.8

m∠2 = 104.5°

m∠1 + 37.1 = m∠2 = 104.5

m∠1  = 67.4°

One evening, Mr. Smith, Sam, and Sydney went to see the Myrtle Beach Pelicans’ Game. They each bought a ticket and a hat that cost $10 each. How much was each ticket to enter the ballpark?

Answers

Answer:

You can't answer this question without knowing the total amount of money they all had in total.

Water is coming out of a fountain is modeled by the function f(x)=-x^2+8x+2 where f(x) represents the height, in feet, if the water from the fountain at different times x, in seconds

Answers

The answer would be 1

please help whoever wins gets brainliest

Answers

Looks like 108 because if we add the missing square the big square would be 12*12 or area equal to 144. We subtract the missing square which is 6 by 6 or 36 area we get 108 ft.
Another way would be cutting it into a rectangle and square.

Janette measures the heights in inches of some of the flowers in her garden to see which ones are growing more. The heights are listed below. Which line plot represents the data correctly? 7 1 2 712 7 7 7 1 2 712 6 3 4 634 7 1 4 714 7 1 4 714 7 1 4 714 7 7 6 1 2 612 7 3 4 734 7 3 4 734 6 3 4 634 8 1 4 814 7 7 7 1 2 712 7 7 7 1 4 714 7 7 7 1 2 712 7 1 2 712

Answers

Answer:

Dot plot A

Step-by-step explanation:

In other obtain the solution to the question given, we obtain the frequency distribution of the data values given and compare to the box plots given to identify which boxplot correctly conveys the data.

___ Frequency

6 1/2 - 1

6 3/4 - 2

7 = 5

7 1/4 = 4

7 1/2 = 5

7 3/4 = 2

8 1/4 = 1

PLease tell me how to do it that will give you brainliest

Answers

Answer: Table H would be the correct answer;

The rule of a function is that for each x-value given there can't be more than 1 y-value

In Table F:

x = -13, then y = -2

x = -13, then y = 0

x = -13, then y = 5

x = -13, then y = 7

For the x-value -13, there are 4 different y-values, so it's not a function.

In Table G:

x = -6, then y = 3

x = -1, then y = -1

x = -1, then y = 5

x = 10, then y = -9

For the x-value -1, there are 2 different y-values, hence this isn't a function.

In Table H:

x = 1, then y = 4

x = 3, then y = 4

x = 7, then y = 4

x = 12, then y = 4

For each x-value, there is only 1 y-value, so this is a function.

In table J:

x = -9, then y = -7

x = -2, then y = -5

x = 0, then y = 0

x = 0, then y = 6

For the x-value 0, there are 2 different y-value therefore this isn't a function

Hope this helps!

What is the quotient of 5.76 x 10^7 and 9.0 x 10^4 expressed in scientific notation?

Answers

Answer:

6.4 x 10²

Step-by-step explanation:

1st step: divide 5.76 by 9 which is .64

2nd step: 10^7÷10^4 = 10^7-4 or 10³

The volume of the figure, round to the nearest hundredth.

Answers

Answer: Imma need a picture to find the volume love.

Step-by-step explanation:

find the cost of 24 piece of shshi from each restaruant

Answers

Answer:

sushi* and there is no picture.

Step-by-step explanation:

Find the volume of a cylınder whose base is a circle with a diameter of 18 inches.

Answers

Answer:

257.48 inches

Step-by-step explanation:

pi * radius^2 * Hight

find radius by dividing the diameter in half

pi = 3.14

What is the answer to this?-

Answers

Answer:

Yes it is congruent by HL

help asap pleaseeee !!!!!!

Answers

Answer:

C. ⅓ × (8 + 7) + 12

Step-by-step explanation:

Let's translate each statement given to numerals and symbols first:

Twelve => 12

One-third => ⅓

Sum of 8 and 7 => (8 + 7)

Representing this statement altogether will be:

⅓ of (8 + 7) = ⅓ × (8 + 7)

If 13 is added, we would get:

⅓ × (8 + 7) + 12

anyone knowww thiusss ?




no scammers pleaseeee

Answers

Answer:

about 9cm

Step-by-step explanation:

Muliply the radius by itself: 16x16

Mulitply the answer by 3.14.

Divide 7252 by the answer to get 9

What is a reasonable domain and range for the function in #13 If the frame must be at least 1 inch and no more than 3 inches wide

Answers

Answer:

[tex]Domain: \{1 \le x \le 3\}[/tex]

Step-by-step explanation:

Given

See attachment for options

Required

Determine the domain and the range

From the question, we understand that:

[tex]x \ge 1[/tex] --- at least 1 inch

[tex]x \le 3[/tex] --- not more than 3 inches

So, the domain is:

[tex]x \ge 1[/tex] and [tex]x \le 3[/tex]

Rewrite as:

[tex]1 \le x[/tex] and [tex]x \le 3[/tex]

Combine both:

[tex]1 \le x \le 3[/tex]

Hence, the domain is:

[tex]Domain: \{1 \le x \le 3\}[/tex]

There is no available to determine the range.

A general rule is that, substitute the limits of the domain (i.e. 1 and 3) in the function formula, the result is the range of the dataset.

For instance:

[tex]A(x) = 5x[/tex]

We have:

[tex]A(1) = 5*1=5[/tex]

[tex]A(3) = 5*3=15[/tex]

The range will be:

[tex]Range: \{5 \le A(x) \le 15\}[/tex]

Jack wants to purchase a DVD player who’s price was $162.50 he has $50 only how much more money does Jack need to purchase the DVD player

Answers

$112.50 is how much jack will need
Jack needs $112.50 to purchase the DVD player

$162.50- 50= 123.50

Unit l2 trigonometry homework 6 law of cosines
Does anyone know the answer please help!​

Answers

Step-by-step explanation:

The law of cosine states that:

[tex]C^2=B^2+A^2-2AB\cos(\theta)[/tex], where C is the side opposite of angle theta, and A and B are the adjacent sides of the angle.

In this case, A is equal to 21, and B is equal to 30 (or it could also be the other way around). That means that:

[tex]C^2=21^2+30^2-2\cdot21\cdot30\cos(73)\\C=\sqrt{21^2+30^2-2\cdot21\cdot30\cos(73)}=\boxed{31.186722}[/tex] (to nearest 6 digits)

Answer:

102.61

Step-by-step explanation:

assuming that QS is the hypothesis we us the fromula like this:

[tex]30cos(73)[/tex]

which gives us 8.7711511417 or we can round to 8.77

assuming that QS is the adjacent  we use:

[tex]\frac{30}{cos(73)}[/tex]

which gives us 102.609108595 or rounded to 102.61

looking at both answers we can see that 102.61 would be the correct answer.

A small toy car costs $3. A large toy car costs 5 times as much as the small one.
Aaron wants to buy one of each. Which equation can he use to find the cost (a)
of the two cars?

Answers

Answer:

(3 x 1) + (3(5) x 1) = a

Step-by-step explanation:

Identify the net that corresponds to the figure below. A. B. C. D.

Answers

Answer:

The net that corresponds to it is ,B

please simple answers i had state testing today

Answers

Answer:

The mode is 10. The range is 6. The IQR is 5.

Step-by-step explanation:

what is the value of x?

Answers

15 ehdhbdbbfbfhfhfbrfhhfhfhfhfhfhfhhfhfhfhfhfhhffjjfh
15 is the answer bdbdjjsjwushs

What is the average rate of change of f(x)=x2+x−6 between x=1 and x=3 ?

A
5

B
–5

C
15

D
10

Answers

Answer:

10

Step-by-step explanation:

Substitute 1 in place of x and the result is -4.

Again substitute 3 in place of x and the result is 6.

Change=6-(-4)=10

A volume of the cone is (1/3) the volume of a cylinder. The volume of the sphere is what fraction of the volume of the cylinder?

Answers

Answer:

4/3

Step-by-step explanation:

To know this, let's write down the formulas for the volume of cylinder and sphere.

Vs = 4/3πr³   (1)

Vc = π r² h    (2)

Now, we do have a little problem here and its the fact that the sphere do not have a height like the cylinder do. But in this case so if you want to have an idea of the fraction of the volume, we will assume that the cylinder has the same height as its radius. Assuming this we have the following:

Vs / Vc = 4πr³ / 3πr²h

Vs/Vc = 4πr³ / 3πr³

From here, we can cancel out the values of π and r³:

Vs/Vc = 4/3

Vs = 4/3 Vc

Thus we can conclude that the volume of the sphere is 4/3 the volume of a cylinder.

Hope this helps

Other Questions
What is the measure of angle L? Round to the nearest hundreth. Help please!! What is the value of tan C? Leave your answer as a fraction.Help please!! 7. A person sits on a freely spinning lab stool (no friction). When this person extends her arms, A) her moment of inertia decreases and her angular velocity increases. B) her moment of inertia decreases and her angular velocity decreases. C) her moment of inertia increases and her angular velocity increases. D) her moment of inertia increases and her angular velocity decreases. E) her moment of inertia decreases and her angular velocity remains the same I NEED HELP ASAPIGNORE THIS TEXT: Ali wants to buy a car that costs $8,000. One option is to use a monthly payment plan. Her monthly payments would be $143.75 over 5 years. The second option is to pay the $8,000 at the time of purchase. About how much more will Ali pay using the monthly payment plan than paying the $8,000 at the time of purchase? The Clean Water Act (CWA) of 1972 did all of the following except _____.take over the EPA's authority to impose pollution control programsnot permit pollutants to be discharged from pipes or man-made ditches into navigable watersregulate pollutants discharged into US watersset water-quality standards Solve the equation 2032 * 93 equals? Qu diremos sobre el tema escogido? ________ probability is based on a uniform probability. Ask yourself: What SHOULD happen when you conduct an experiment? A) Simple B) TheoreticalC) Experimental _____ probability is based on relative frequency. Ask yourself: What ACTUALLY occurred during such an experiment? A) Simple B) Theoretical C) Experimental HELPP!!!! PLEASE ANSWER ASAP WILL GIVE YOU BRAINLIEST Write the number 3.8 x 10-4 in standard form. What is the length of BC? If the answer is not an integer, leave it in simplest radical form. The diagram is not drawn to scale.O A. 13/2 ftOB. 135 ftC. 26 ftD. 13 ft Can someone plz help me with this one problem plz!!! 1.What happened at Jackson's inauguration? Explain The rapid melting of Greenland's ice cap could disrupt the North Atlantic Deep Water (NADW) formation by ________. adding excess nutrients to cold northern waters acidifying ocean waters of the North Atlantic adding heat and salt to deep ocean waters in the North Atlantic adding huge amounts of dense fresh water to the deep ocean in the North Atlantic adding huge amounts of less dense fresh water to the surface of the North Atlantic kiras best distance in the long jump is 3 7/10 meters. Which decimal is equivalent to 3 7/10?help me pleaseee What is the answer please tell me help please no links! thank you A game is played by spinning a wheel that contains the words Win and Lose. The wheel has 10 slots, with Win written four times and Lose written six times. If the wheel lands on win, you win $5; on lose, you win nothing. It costs $2 to play. What is the expected value?A) $1B) $2C) $0D) -$2 Based on the information in the graph, in which month do 24% of the students have a birthday? Can someone please help me with science.